LSAT and Law School Admissions Forum

Get expert LSAT preparation and law school admissions advice from PowerScore Test Preparation.

User avatar
 Dave Killoran
PowerScore Staff
  • PowerScore Staff
  • Posts: 5853
  • Joined: Mar 25, 2011
|
#59736
Complete Question Explanation
(The complete setup for this game can be found here: lsat/viewtopic.php?t=1753)

The correct answer choice is (A)

If L is within exactly three areas, L cannot be in area S (if L were in S, then it could only be in S and T) and therefore L must be in areas R, U, and T:

pt15_j95_g2_q13.png
Since L and M cannot be in the same detection areas, M cannot be in areas R, U, and T, and therefore answer choices (D) and (E) are incorrect. Since M must be in at least one detection area, it follows that M must be in area S. According to the Not Laws in our initial setup, K can never be in area S, and therefore answer choice (C) is incorrect. In answer choice (B), it is impossible for J to be in three areas since any plane in area S cannot be in area R and U (and plane J is in area S from the second rule). Thus, J can only be in at most two areas. Since answer choice (B) is incorrect, answer choice (A) is proven correct by process of elimination.
You do not have the required permissions to view the files attached to this post.

Get the most out of your LSAT Prep Plus subscription.

Analyze and track your performance with our Testing and Analytics Package.